Are the partial sums for $sum_{n=1}^{infty}sin(n^a)$ bounded for $ageq1$ and unbounded for $0<a<1$?
up vote
2
down vote
favorite
I know that the partial sums of $$sum_{n=1}^{infty}sin(n)$$
are bounded between $frac{cosleft(frac{1}{2}right)-1}{2sinleft(frac{1}{2} right)}$ and $frac{1+cosleft(frac{1}{2} right)}{2sinleft(frac{1}{2}right)}$.
On the other hand, the partial sums of
$$sum_{n=1}^{infty}sin(sqrt{n})$$
are unbounded.
I think that the partial sums of $sum_{n=1}^{infty}sin(n^a)$ are bounded for $a geq 1$ and unbounded for $0< a<1$, but how can I prove this? I think this question involves Euler-Maclaurin sum.
calculus sequences-and-series trigonometric-series euler-maclaurin
|
show 4 more comments
up vote
2
down vote
favorite
I know that the partial sums of $$sum_{n=1}^{infty}sin(n)$$
are bounded between $frac{cosleft(frac{1}{2}right)-1}{2sinleft(frac{1}{2} right)}$ and $frac{1+cosleft(frac{1}{2} right)}{2sinleft(frac{1}{2}right)}$.
On the other hand, the partial sums of
$$sum_{n=1}^{infty}sin(sqrt{n})$$
are unbounded.
I think that the partial sums of $sum_{n=1}^{infty}sin(n^a)$ are bounded for $a geq 1$ and unbounded for $0< a<1$, but how can I prove this? I think this question involves Euler-Maclaurin sum.
calculus sequences-and-series trigonometric-series euler-maclaurin
You mean the partial sums of these series are bounded/unbounded.
– zhw.
Nov 10 at 20:12
1
It would be good to edit to reflect this.
– zhw.
Nov 10 at 20:26
1
The partial sums of $sum sin(n^alpha)$ are not bounded for $alpha>1$, but $sum_{n=1}^{N}sin(n^alpha)$ can be suitably controlled by a power of $N$ times a power of $log N$. See Weyl's inequality, giving $$sum_{n=1}^{N}sin(n^2) ll sqrt{N}log^2 N,$$ for instance.
– Jack D'Aurizio
Nov 10 at 22:11
1
If you could achieve such bound $sqrt{N}log^2 N$, it solves your first question in math.stackexchange.com/questions/215528/…. When I tried it years ago, it did not work out very well, I could obtain $alphaleq 7/8$. If you managed to prove such bound, please include your method as an answer.
– i707107
Nov 11 at 3:51
1
@JackD'Aurizio You mentioned that the partial sums are unbounded when $a>1$. Would you please include that as an answer? I know Terry Tao solved for $a>1$ integer case in here mathoverflow.net/questions/201250/…
– i707107
Nov 12 at 18:14
|
show 4 more comments
up vote
2
down vote
favorite
up vote
2
down vote
favorite
I know that the partial sums of $$sum_{n=1}^{infty}sin(n)$$
are bounded between $frac{cosleft(frac{1}{2}right)-1}{2sinleft(frac{1}{2} right)}$ and $frac{1+cosleft(frac{1}{2} right)}{2sinleft(frac{1}{2}right)}$.
On the other hand, the partial sums of
$$sum_{n=1}^{infty}sin(sqrt{n})$$
are unbounded.
I think that the partial sums of $sum_{n=1}^{infty}sin(n^a)$ are bounded for $a geq 1$ and unbounded for $0< a<1$, but how can I prove this? I think this question involves Euler-Maclaurin sum.
calculus sequences-and-series trigonometric-series euler-maclaurin
I know that the partial sums of $$sum_{n=1}^{infty}sin(n)$$
are bounded between $frac{cosleft(frac{1}{2}right)-1}{2sinleft(frac{1}{2} right)}$ and $frac{1+cosleft(frac{1}{2} right)}{2sinleft(frac{1}{2}right)}$.
On the other hand, the partial sums of
$$sum_{n=1}^{infty}sin(sqrt{n})$$
are unbounded.
I think that the partial sums of $sum_{n=1}^{infty}sin(n^a)$ are bounded for $a geq 1$ and unbounded for $0< a<1$, but how can I prove this? I think this question involves Euler-Maclaurin sum.
calculus sequences-and-series trigonometric-series euler-maclaurin
calculus sequences-and-series trigonometric-series euler-maclaurin
edited Nov 10 at 22:22
zhw.
70.4k42975
70.4k42975
asked Nov 10 at 20:05
Larry
1,1021622
1,1021622
You mean the partial sums of these series are bounded/unbounded.
– zhw.
Nov 10 at 20:12
1
It would be good to edit to reflect this.
– zhw.
Nov 10 at 20:26
1
The partial sums of $sum sin(n^alpha)$ are not bounded for $alpha>1$, but $sum_{n=1}^{N}sin(n^alpha)$ can be suitably controlled by a power of $N$ times a power of $log N$. See Weyl's inequality, giving $$sum_{n=1}^{N}sin(n^2) ll sqrt{N}log^2 N,$$ for instance.
– Jack D'Aurizio
Nov 10 at 22:11
1
If you could achieve such bound $sqrt{N}log^2 N$, it solves your first question in math.stackexchange.com/questions/215528/…. When I tried it years ago, it did not work out very well, I could obtain $alphaleq 7/8$. If you managed to prove such bound, please include your method as an answer.
– i707107
Nov 11 at 3:51
1
@JackD'Aurizio You mentioned that the partial sums are unbounded when $a>1$. Would you please include that as an answer? I know Terry Tao solved for $a>1$ integer case in here mathoverflow.net/questions/201250/…
– i707107
Nov 12 at 18:14
|
show 4 more comments
You mean the partial sums of these series are bounded/unbounded.
– zhw.
Nov 10 at 20:12
1
It would be good to edit to reflect this.
– zhw.
Nov 10 at 20:26
1
The partial sums of $sum sin(n^alpha)$ are not bounded for $alpha>1$, but $sum_{n=1}^{N}sin(n^alpha)$ can be suitably controlled by a power of $N$ times a power of $log N$. See Weyl's inequality, giving $$sum_{n=1}^{N}sin(n^2) ll sqrt{N}log^2 N,$$ for instance.
– Jack D'Aurizio
Nov 10 at 22:11
1
If you could achieve such bound $sqrt{N}log^2 N$, it solves your first question in math.stackexchange.com/questions/215528/…. When I tried it years ago, it did not work out very well, I could obtain $alphaleq 7/8$. If you managed to prove such bound, please include your method as an answer.
– i707107
Nov 11 at 3:51
1
@JackD'Aurizio You mentioned that the partial sums are unbounded when $a>1$. Would you please include that as an answer? I know Terry Tao solved for $a>1$ integer case in here mathoverflow.net/questions/201250/…
– i707107
Nov 12 at 18:14
You mean the partial sums of these series are bounded/unbounded.
– zhw.
Nov 10 at 20:12
You mean the partial sums of these series are bounded/unbounded.
– zhw.
Nov 10 at 20:12
1
1
It would be good to edit to reflect this.
– zhw.
Nov 10 at 20:26
It would be good to edit to reflect this.
– zhw.
Nov 10 at 20:26
1
1
The partial sums of $sum sin(n^alpha)$ are not bounded for $alpha>1$, but $sum_{n=1}^{N}sin(n^alpha)$ can be suitably controlled by a power of $N$ times a power of $log N$. See Weyl's inequality, giving $$sum_{n=1}^{N}sin(n^2) ll sqrt{N}log^2 N,$$ for instance.
– Jack D'Aurizio
Nov 10 at 22:11
The partial sums of $sum sin(n^alpha)$ are not bounded for $alpha>1$, but $sum_{n=1}^{N}sin(n^alpha)$ can be suitably controlled by a power of $N$ times a power of $log N$. See Weyl's inequality, giving $$sum_{n=1}^{N}sin(n^2) ll sqrt{N}log^2 N,$$ for instance.
– Jack D'Aurizio
Nov 10 at 22:11
1
1
If you could achieve such bound $sqrt{N}log^2 N$, it solves your first question in math.stackexchange.com/questions/215528/…. When I tried it years ago, it did not work out very well, I could obtain $alphaleq 7/8$. If you managed to prove such bound, please include your method as an answer.
– i707107
Nov 11 at 3:51
If you could achieve such bound $sqrt{N}log^2 N$, it solves your first question in math.stackexchange.com/questions/215528/…. When I tried it years ago, it did not work out very well, I could obtain $alphaleq 7/8$. If you managed to prove such bound, please include your method as an answer.
– i707107
Nov 11 at 3:51
1
1
@JackD'Aurizio You mentioned that the partial sums are unbounded when $a>1$. Would you please include that as an answer? I know Terry Tao solved for $a>1$ integer case in here mathoverflow.net/questions/201250/…
– i707107
Nov 12 at 18:14
@JackD'Aurizio You mentioned that the partial sums are unbounded when $a>1$. Would you please include that as an answer? I know Terry Tao solved for $a>1$ integer case in here mathoverflow.net/questions/201250/…
– i707107
Nov 12 at 18:14
|
show 4 more comments
1 Answer
1
active
oldest
votes
up vote
1
down vote
accepted
If $a>1$ is an integer, then this post in MO by Terry Tao solves that the partial sums are not bounded.
If $a>1$ is not an integer, the same argument by Terry Tao still works, since we have equidistribution of $n^a$ mod $2pi$, and any sum or difference of $(n+i)^a$ with $1leq i leq h$ modulo $2pi$. Let $X_i$ be the random variable $sin^a (k+i)$ where $k=1,ldots n$. Assuming the boundedness of partial sums, we end up having a contradiction that the random variables $X_1, ldots, X_h$ such that $mathrm{Var}(X_1+cdots+X_h)$ is bounded as $hrightarrow infty$ by assumption, but $mathrm{Var}(X_1+cdots+X_h)sim h/2$ as $hrightarrowinfty$.
add a comment |
1 Answer
1
active
oldest
votes
1 Answer
1
active
oldest
votes
active
oldest
votes
active
oldest
votes
up vote
1
down vote
accepted
If $a>1$ is an integer, then this post in MO by Terry Tao solves that the partial sums are not bounded.
If $a>1$ is not an integer, the same argument by Terry Tao still works, since we have equidistribution of $n^a$ mod $2pi$, and any sum or difference of $(n+i)^a$ with $1leq i leq h$ modulo $2pi$. Let $X_i$ be the random variable $sin^a (k+i)$ where $k=1,ldots n$. Assuming the boundedness of partial sums, we end up having a contradiction that the random variables $X_1, ldots, X_h$ such that $mathrm{Var}(X_1+cdots+X_h)$ is bounded as $hrightarrow infty$ by assumption, but $mathrm{Var}(X_1+cdots+X_h)sim h/2$ as $hrightarrowinfty$.
add a comment |
up vote
1
down vote
accepted
If $a>1$ is an integer, then this post in MO by Terry Tao solves that the partial sums are not bounded.
If $a>1$ is not an integer, the same argument by Terry Tao still works, since we have equidistribution of $n^a$ mod $2pi$, and any sum or difference of $(n+i)^a$ with $1leq i leq h$ modulo $2pi$. Let $X_i$ be the random variable $sin^a (k+i)$ where $k=1,ldots n$. Assuming the boundedness of partial sums, we end up having a contradiction that the random variables $X_1, ldots, X_h$ such that $mathrm{Var}(X_1+cdots+X_h)$ is bounded as $hrightarrow infty$ by assumption, but $mathrm{Var}(X_1+cdots+X_h)sim h/2$ as $hrightarrowinfty$.
add a comment |
up vote
1
down vote
accepted
up vote
1
down vote
accepted
If $a>1$ is an integer, then this post in MO by Terry Tao solves that the partial sums are not bounded.
If $a>1$ is not an integer, the same argument by Terry Tao still works, since we have equidistribution of $n^a$ mod $2pi$, and any sum or difference of $(n+i)^a$ with $1leq i leq h$ modulo $2pi$. Let $X_i$ be the random variable $sin^a (k+i)$ where $k=1,ldots n$. Assuming the boundedness of partial sums, we end up having a contradiction that the random variables $X_1, ldots, X_h$ such that $mathrm{Var}(X_1+cdots+X_h)$ is bounded as $hrightarrow infty$ by assumption, but $mathrm{Var}(X_1+cdots+X_h)sim h/2$ as $hrightarrowinfty$.
If $a>1$ is an integer, then this post in MO by Terry Tao solves that the partial sums are not bounded.
If $a>1$ is not an integer, the same argument by Terry Tao still works, since we have equidistribution of $n^a$ mod $2pi$, and any sum or difference of $(n+i)^a$ with $1leq i leq h$ modulo $2pi$. Let $X_i$ be the random variable $sin^a (k+i)$ where $k=1,ldots n$. Assuming the boundedness of partial sums, we end up having a contradiction that the random variables $X_1, ldots, X_h$ such that $mathrm{Var}(X_1+cdots+X_h)$ is bounded as $hrightarrow infty$ by assumption, but $mathrm{Var}(X_1+cdots+X_h)sim h/2$ as $hrightarrowinfty$.
edited Nov 14 at 5:17
answered Nov 14 at 5:09
i707107
11.8k21447
11.8k21447
add a comment |
add a comment |
Sign up or log in
StackExchange.ready(function () {
StackExchange.helpers.onClickDraftSave('#login-link');
});
Sign up using Google
Sign up using Facebook
Sign up using Email and Password
Post as a guest
Required, but never shown
StackExchange.ready(
function () {
StackExchange.openid.initPostLogin('.new-post-login', 'https%3a%2f%2fmath.stackexchange.com%2fquestions%2f2993073%2fare-the-partial-sums-for-sum-n-1-infty-sinna-bounded-for-a-geq1-and%23new-answer', 'question_page');
}
);
Post as a guest
Required, but never shown
Sign up or log in
StackExchange.ready(function () {
StackExchange.helpers.onClickDraftSave('#login-link');
});
Sign up using Google
Sign up using Facebook
Sign up using Email and Password
Post as a guest
Required, but never shown
Sign up or log in
StackExchange.ready(function () {
StackExchange.helpers.onClickDraftSave('#login-link');
});
Sign up using Google
Sign up using Facebook
Sign up using Email and Password
Post as a guest
Required, but never shown
Sign up or log in
StackExchange.ready(function () {
StackExchange.helpers.onClickDraftSave('#login-link');
});
Sign up using Google
Sign up using Facebook
Sign up using Email and Password
Sign up using Google
Sign up using Facebook
Sign up using Email and Password
Post as a guest
Required, but never shown
Required, but never shown
Required, but never shown
Required, but never shown
Required, but never shown
Required, but never shown
Required, but never shown
Required, but never shown
Required, but never shown
You mean the partial sums of these series are bounded/unbounded.
– zhw.
Nov 10 at 20:12
1
It would be good to edit to reflect this.
– zhw.
Nov 10 at 20:26
1
The partial sums of $sum sin(n^alpha)$ are not bounded for $alpha>1$, but $sum_{n=1}^{N}sin(n^alpha)$ can be suitably controlled by a power of $N$ times a power of $log N$. See Weyl's inequality, giving $$sum_{n=1}^{N}sin(n^2) ll sqrt{N}log^2 N,$$ for instance.
– Jack D'Aurizio
Nov 10 at 22:11
1
If you could achieve such bound $sqrt{N}log^2 N$, it solves your first question in math.stackexchange.com/questions/215528/…. When I tried it years ago, it did not work out very well, I could obtain $alphaleq 7/8$. If you managed to prove such bound, please include your method as an answer.
– i707107
Nov 11 at 3:51
1
@JackD'Aurizio You mentioned that the partial sums are unbounded when $a>1$. Would you please include that as an answer? I know Terry Tao solved for $a>1$ integer case in here mathoverflow.net/questions/201250/…
– i707107
Nov 12 at 18:14